LSAT and Law School Admissions Forum

Get expert LSAT preparation and law school admissions advice from PowerScore Test Preparation.

 Administrator
PowerScore Staff
  • PowerScore Staff
  • Posts: 8917
  • Joined: Feb 02, 2011
|
#36270
Complete Question Explanation

Weaken. The correct answer choice is (C)

The literary critic points out that often the heirs of a writer will publish much of that writer’s
remaining work, regardless of merit. Because many writers possess manuscripts they consider
unworthy and would not want published, the critic concludes that a successful writer who decides
not to publish a recently completed manuscript should destroy it immediately.

Note that the last word of the stimulus—immediately—is rather extreme (remember, always look for
words that seems absolute or overly strong for the situation). You may have had a strong reaction to
the critic’s recommendation because of that extremity, and that reaction is relevant to the fl aw in the
critic’s reasoning. Destroying a manuscript immediately seems like a severe step, and one that does
not allow for any reassessment of the work. Over time, there might be many benefi ts to be derived
from keeping an initially dissatisfactory manuscript (especially if the probability of immediate death
is low, as would be the case for most authors).

Since you are asked to undermine the critic’s reasoning, you should seek an answer choice that
suggests that writers might not want to destroy their completed manuscripts immediately.

Answer choice (A): The argument is about successful writers, and the suggestion that successful
living writers should immediately destroy works that they decide not to publish. This answer choice
is about writers who become successful after they die, so even when this answer is taken as true it
does not impact the argument made by the critic (once you are dead, you can’t go back and destroy
your manuscripts, obviously).

Answer choice (B): This is a classic Shell Game answer. The stimulus specifi cally discusses
manuscripts and whether they should be destroyed. This answer, on the other hand, discusses
personal correspondence, and the two are not the same.

Answer choice (C): This is the correct answer choice. If many successful writers will change
their views of their recently completed works, then those writers might come to see their initial
assessments as overly critical and then choose to publish a manuscript. That eventuality could not
occur if they had immediately destroyed the manuscript upon completion, so this choice represents
an effective attack on the critic’s argument.

Answer choice (D): This is the most frequently selected incorrect answer. The key to avoiding
this answer is to understand that many of the posthumously published books were judged worthy
by the author prior to his or her death. For example, say that an author completed a manuscript
and determined it was worthy of publication, but then died before the book was published. Does
the existence of this scenario undermine the critic’s argument? No, because the critic argues that
manuscripts that are not going to be published should be destroyed, and the manuscripts in the
answer choice do not fall into that category.

Answer choice (E): If anything, this answer agrees with the critic since it suggests that the heirs of a
successful writer will not be able to discern a quality work from an inferior work. According to the
argument, the heirs publish the works regardless of merit, so whether they deem themselves qualifi ed
to judge the works is not relevant.
 eober
  • Posts: 107
  • Joined: Jul 24, 2014
|
#16307
Hi,

In weaken questions I tend to have difficulty eliminating an answer choice because more than one looks like it will weaken the argument. In this question I was also considering answer choice D. Answer choice C makes perfect sense as well but how do we eliminate answer choice D.

Also, do you have a recommendation on what to do in a case where both answers choices seems to weaken the argument, or what should I focus on in a weaken question?

Thank you !
 David Boyle
PowerScore Staff
  • PowerScore Staff
  • Posts: 836
  • Joined: Jun 07, 2013
|
#16335
eober wrote:Hi,

In weaken questions I tend to have difficulty eliminating an answer choice because more than one looks like it will weaken the argument. In this question I was also considering answer choice D. Answer choice C makes perfect sense as well but how do we eliminate answer choice D.

Also, do you have a recommendation on what to do in a case where both answers choices seems to weaken the argument, or what should I focus on in a weaken question?

Thank you !
Hello eober,

"Many", in answer D, is much weaker than "most" which is in answer C. If you have a million objects, a thousand may be "many" of them, but still only one-thousandth of the total.
"what should I focus on in a weaken question?" is a broad question. Obviously, focusing on the conclusion is important, and focusing on any causality issues. You may want to revisit the theory in chapter 3 for more details.
If both answer choices seem to weaken, then look carefully again: one choice should weaken much more than the other does.

Hope this helps,
David
 eober
  • Posts: 107
  • Joined: Jul 24, 2014
|
#16350
Hi,

So we eliminated answer choice D because "many" is weaker, but not because of another reason. Answer choice C is better at weakening? Did I understand that right?
 Emily Haney-Caron
PowerScore Staff
  • PowerScore Staff
  • Posts: 577
  • Joined: Jan 12, 2012
|
#16388
Hi eober,

Basically, just because "many" posthumously published books would have been published by the author anyway, that doesn't mean that any of those are ones the authors would have deemed unworthy of publication. Therefore, D is out. For answer choice C, this actually directly weakens it by suggesting that even if the authors dislike it right after completing it, they should not destroy it because they actually may come to like it later.

Get the most out of your LSAT Prep Plus subscription.

Analyze and track your performance with our Testing and Analytics Package.